How to prove that $mathbb{Z}[(1+sqrt{2})/2]$ is a Euclidean domain?












1












$begingroup$


I know how to prove Euclidean domain for $mathbb{Z}[sqrt{2}]$ and $mathbb{Z}[sqrt{-2}]$, etc, but I am getting stuck on this notation. From what I've read,



We let $R_d$ be the ring $mathbb{Z}[(1+sqrt{2})/2]$ defined as $R_d={x + yw : x ,y ∈ mathbb{Z}}$, where $w = sqrt{d}$ if $d$ is not equivalent to $1$ mod $4$, and $(1+sqrt{d})/2$ if $d$ is equivalent to $1$ mod $4$. So clearly, in this problem, we are assuming that $d= 2$ is equiv. to $1$ mod $4$. But, I really don't know what to do with this information. Does the norm change? What is the norm? Is it the same as it would be for $sqrt{2}$? I just need a little more information to get going, and I would really appreciate any help you're willing to provide. Thank you.










share|cite|improve this question











$endgroup$








  • 1




    $begingroup$
    "So clearly, in this problem, we are assuming that $d=2$ is equiv. to $1$ mod $4$." But $2$ is not equivalent to $1$ mod $4$, so you are in the first case and should be considering $mathbb{Z}[sqrt{2}]$.
    $endgroup$
    – André 3000
    Nov 30 '18 at 6:27


















1












$begingroup$


I know how to prove Euclidean domain for $mathbb{Z}[sqrt{2}]$ and $mathbb{Z}[sqrt{-2}]$, etc, but I am getting stuck on this notation. From what I've read,



We let $R_d$ be the ring $mathbb{Z}[(1+sqrt{2})/2]$ defined as $R_d={x + yw : x ,y ∈ mathbb{Z}}$, where $w = sqrt{d}$ if $d$ is not equivalent to $1$ mod $4$, and $(1+sqrt{d})/2$ if $d$ is equivalent to $1$ mod $4$. So clearly, in this problem, we are assuming that $d= 2$ is equiv. to $1$ mod $4$. But, I really don't know what to do with this information. Does the norm change? What is the norm? Is it the same as it would be for $sqrt{2}$? I just need a little more information to get going, and I would really appreciate any help you're willing to provide. Thank you.










share|cite|improve this question











$endgroup$








  • 1




    $begingroup$
    "So clearly, in this problem, we are assuming that $d=2$ is equiv. to $1$ mod $4$." But $2$ is not equivalent to $1$ mod $4$, so you are in the first case and should be considering $mathbb{Z}[sqrt{2}]$.
    $endgroup$
    – André 3000
    Nov 30 '18 at 6:27
















1












1








1


0



$begingroup$


I know how to prove Euclidean domain for $mathbb{Z}[sqrt{2}]$ and $mathbb{Z}[sqrt{-2}]$, etc, but I am getting stuck on this notation. From what I've read,



We let $R_d$ be the ring $mathbb{Z}[(1+sqrt{2})/2]$ defined as $R_d={x + yw : x ,y ∈ mathbb{Z}}$, where $w = sqrt{d}$ if $d$ is not equivalent to $1$ mod $4$, and $(1+sqrt{d})/2$ if $d$ is equivalent to $1$ mod $4$. So clearly, in this problem, we are assuming that $d= 2$ is equiv. to $1$ mod $4$. But, I really don't know what to do with this information. Does the norm change? What is the norm? Is it the same as it would be for $sqrt{2}$? I just need a little more information to get going, and I would really appreciate any help you're willing to provide. Thank you.










share|cite|improve this question











$endgroup$




I know how to prove Euclidean domain for $mathbb{Z}[sqrt{2}]$ and $mathbb{Z}[sqrt{-2}]$, etc, but I am getting stuck on this notation. From what I've read,



We let $R_d$ be the ring $mathbb{Z}[(1+sqrt{2})/2]$ defined as $R_d={x + yw : x ,y ∈ mathbb{Z}}$, where $w = sqrt{d}$ if $d$ is not equivalent to $1$ mod $4$, and $(1+sqrt{d})/2$ if $d$ is equivalent to $1$ mod $4$. So clearly, in this problem, we are assuming that $d= 2$ is equiv. to $1$ mod $4$. But, I really don't know what to do with this information. Does the norm change? What is the norm? Is it the same as it would be for $sqrt{2}$? I just need a little more information to get going, and I would really appreciate any help you're willing to provide. Thank you.







abstract-algebra ring-theory euclidean-domain






share|cite|improve this question















share|cite|improve this question













share|cite|improve this question




share|cite|improve this question








edited Nov 30 '18 at 6:27









André 3000

12.6k22243




12.6k22243










asked Nov 30 '18 at 4:42









Zoë SZoë S

61




61








  • 1




    $begingroup$
    "So clearly, in this problem, we are assuming that $d=2$ is equiv. to $1$ mod $4$." But $2$ is not equivalent to $1$ mod $4$, so you are in the first case and should be considering $mathbb{Z}[sqrt{2}]$.
    $endgroup$
    – André 3000
    Nov 30 '18 at 6:27
















  • 1




    $begingroup$
    "So clearly, in this problem, we are assuming that $d=2$ is equiv. to $1$ mod $4$." But $2$ is not equivalent to $1$ mod $4$, so you are in the first case and should be considering $mathbb{Z}[sqrt{2}]$.
    $endgroup$
    – André 3000
    Nov 30 '18 at 6:27










1




1




$begingroup$
"So clearly, in this problem, we are assuming that $d=2$ is equiv. to $1$ mod $4$." But $2$ is not equivalent to $1$ mod $4$, so you are in the first case and should be considering $mathbb{Z}[sqrt{2}]$.
$endgroup$
– André 3000
Nov 30 '18 at 6:27






$begingroup$
"So clearly, in this problem, we are assuming that $d=2$ is equiv. to $1$ mod $4$." But $2$ is not equivalent to $1$ mod $4$, so you are in the first case and should be considering $mathbb{Z}[sqrt{2}]$.
$endgroup$
– André 3000
Nov 30 '18 at 6:27












0






active

oldest

votes











Your Answer





StackExchange.ifUsing("editor", function () {
return StackExchange.using("mathjaxEditing", function () {
StackExchange.MarkdownEditor.creationCallbacks.add(function (editor, postfix) {
StackExchange.mathjaxEditing.prepareWmdForMathJax(editor, postfix, [["$", "$"], ["\\(","\\)"]]);
});
});
}, "mathjax-editing");

StackExchange.ready(function() {
var channelOptions = {
tags: "".split(" "),
id: "69"
};
initTagRenderer("".split(" "), "".split(" "), channelOptions);

StackExchange.using("externalEditor", function() {
// Have to fire editor after snippets, if snippets enabled
if (StackExchange.settings.snippets.snippetsEnabled) {
StackExchange.using("snippets", function() {
createEditor();
});
}
else {
createEditor();
}
});

function createEditor() {
StackExchange.prepareEditor({
heartbeatType: 'answer',
autoActivateHeartbeat: false,
convertImagesToLinks: true,
noModals: true,
showLowRepImageUploadWarning: true,
reputationToPostImages: 10,
bindNavPrevention: true,
postfix: "",
imageUploader: {
brandingHtml: "Powered by u003ca class="icon-imgur-white" href="https://imgur.com/"u003eu003c/au003e",
contentPolicyHtml: "User contributions licensed under u003ca href="https://creativecommons.org/licenses/by-sa/3.0/"u003ecc by-sa 3.0 with attribution requiredu003c/au003e u003ca href="https://stackoverflow.com/legal/content-policy"u003e(content policy)u003c/au003e",
allowUrls: true
},
noCode: true, onDemand: true,
discardSelector: ".discard-answer"
,immediatelyShowMarkdownHelp:true
});


}
});














draft saved

draft discarded


















StackExchange.ready(
function () {
StackExchange.openid.initPostLogin('.new-post-login', 'https%3a%2f%2fmath.stackexchange.com%2fquestions%2f3019656%2fhow-to-prove-that-mathbbz1-sqrt2-2-is-a-euclidean-domain%23new-answer', 'question_page');
}
);

Post as a guest















Required, but never shown

























0






active

oldest

votes








0






active

oldest

votes









active

oldest

votes






active

oldest

votes
















draft saved

draft discarded




















































Thanks for contributing an answer to Mathematics Stack Exchange!


  • Please be sure to answer the question. Provide details and share your research!

But avoid



  • Asking for help, clarification, or responding to other answers.

  • Making statements based on opinion; back them up with references or personal experience.


Use MathJax to format equations. MathJax reference.


To learn more, see our tips on writing great answers.




draft saved


draft discarded














StackExchange.ready(
function () {
StackExchange.openid.initPostLogin('.new-post-login', 'https%3a%2f%2fmath.stackexchange.com%2fquestions%2f3019656%2fhow-to-prove-that-mathbbz1-sqrt2-2-is-a-euclidean-domain%23new-answer', 'question_page');
}
);

Post as a guest















Required, but never shown





















































Required, but never shown














Required, but never shown












Required, but never shown







Required, but never shown

































Required, but never shown














Required, but never shown












Required, but never shown







Required, but never shown







Popular posts from this blog

Biblatex bibliography style without URLs when DOI exists (in Overleaf with Zotero bibliography)

ComboBox Display Member on multiple fields

Is it possible to collect Nectar points via Trainline?